IA test 2

Pataasin ang iyong marka sa homework at exams ngayon gamit ang Quizwiz!

An internal audit function is charged with evaluating the compliance of the organization's human resources function with applicable laws, regulations, and internal policies. Which objective(s) is (are) appropriate for this engagement plan? 1. Evaluate compliance with residency policy. 2. Ensure that applicant pools are representative of the population. 3. Assess the process used for responding to employee complaints. 4. Question recently hired employees to assess compliance with interviewing standards. A. 1 and 2 only. B. 1 and 3 only. C. 3 only. D. 3 and 4 only.

B. 1 and 3 only.

An internal auditor is conducting an audit of a contract to build a new branch office. The auditor should consider whether the 1. Materials used in construction meet specified contractual standards. 2. Contractor has established a fraud hotline. 3. Construction is on schedule. A. 1 and 2 only. B. 1 and 3 only. C. 2 and 3 only. D. 1, 2, and 3.

B. 1 and 3 only.

A specific objective of an audit of a company's expenditure cycle is to determine whether all goods paid for have been received and charged to the correct account. This objective addresses which of the following primary objectives identified in the Standards? 1. Reliability and integrity of financial and operational information. 2. Compliance with laws, regulations, policies, procedures, and contracts. 3. Effectiveness and efficiency of operations and programs. 4. Safeguarding of assets. A. 1 and 2 only. B. 1 and 4 only. C. 1, 2, and 4 only. D. 2, 3, and 4 only.

B. 1 and 4 only.

Compliance programs most directly assist organizations by doing which of the following? 1. Developing a plan for business continuity management. 2. Determining director and officer liability. 3. Planning for disaster recovery. A. 1 only. B. 2 only. C. 1 and 2 only. D. 1, 2, and 3.

B. 2 only.

When developing the internal audit plan, the chief audit executive must consider the following expectations of 1. Department managers 2. Stakeholders 3. Human resource managers A. 1 only. B. 2 only. C. 3 only. D. 2 and 3.

B. 2 only.

Which of the following actions by the internal audit activity is (are) appropriate in response to a risk assessment? 1. Although input of senior management and the board should be obtained, the chief audit executive does not need to consider it when developing the internal audit activity's plan of engagements. 2. The high-risk areas should be integrated into an audit plan along with the high-priority requests of management and the audit committee. 3. The risk analysis should be used in determining an audit plan. Thus, it should be performed only on an annual basis. A. 1 only. B. 2 only. C. 1 and 3 only. D. 1 and 2 only.

B. 2 only.

Which of the following represent(s) appropriate internal audit action in response to the risk assessment process? 1. The low-risk areas may be delegated to the external auditor, but the high-risk areas should be performed by the internal audit activity. 2. The high-risk areas should be integrated into an audit work schedule along with the high-priority requests of senior management and the audit committee. 3. The risk analysis should be used in determining an annual audit work schedule. Thus, the risk analysis should be performed only on an annual basis. A. 1 only. B. 2 only. C. 3 only. D. 1 and 3 only.

B. 2 only.

The chief audit executive for an organization has just completed a risk assessment process, identified the areas with the highest risks, and assigned an engagement priority to each. Which of the following conclusions most logically follow(s) from such a risk assessment? 1. Items should be quantified as to risk in the rank order of quantifiable monetary exposure to the organization. 2. The risk priorities should be in order of major control deficiencies. 3. The risk assessment process, though quantified, is the result of professional judgments about both exposures and probability of occurrences. A. 1 only. B. 3 only. C. 2 and 3 only. D. 1, 2, and 3.

B. 3 only.

Management is exploring different ways of reducing or preventing pollution in manufacturing operations. The objective of a pollution prevention audit is to identify opportunities to minimize waste and eliminate pollution at the source. In what order should the following opportunities to reduce waste be considered? 1. Recycling and reuse 2. Elimination at the source 3. Energy conservation 4. Recovery as a usable product 5. Treatment A. 5, 2, 4, 1, and 3. B. 4, 2, 1, 3, and 5. C. 1, 3, 4, 2, and 5. D. 3, 4, 2, 5, and 1.

B. 4, 2, 1, 3, and 5.

A standard engagement work program is not appropriate for which situation? A. A stable operating environment undergoing only minimal changes. B. A complex or changing operating environment. C. Multiple locations with similar operations. D. Subsequent engagements to provide assurance about inventory performed at same location.

B. A complex or changing operating environment.

The advantage attributed to the establishment of internal auditing field offices for work at foreign locations is best described as A. The possibility of increased objectivity of personnel assigned to a field office. B. A reduction of travel time and related travel expense. C. The increased ease of maintaining uniform organization-wide standards. D. More contact with senior personnel leading to an increase in control.

B. A reduction of travel time and related travel expense.

Policies and procedures must be established to guide the internal audit activity. Which of the following statements is false with respect to this requirement? A. The form and content of written policies and procedures depend on the size of the internal audit activity. B. All internal audit activities must have a detailed policies and procedures manual. C. Formal administrative and technical manuals may not be needed by all internal audit activities. D. A small internal audit activity may be managed informally through close supervision and memoranda.

B. All internal audit activities must have a detailed policies and procedures manual.

Which of the following best describes an internal auditor's initial responsibility regarding errors uncovered during a financial statement audit? A. Report the material errors. B. Assess the risk of misrepresentation. C. Discuss the situation with the engagement client. D. Inform the audit committee.

B. Assess the risk of misrepresentation.

George is the new internal auditor for XYZ Corporation. George was in charge of payroll for XYZ just 10 months ago. Performing what services in regard to payroll is considered an impairment of independence or objectivity if performed by George? A. Consulting services. B. Assurance services. C. Assurance or consulting services. D. Neither assurance nor consulting services.

B. Assurance services.

Before an assurance engagement can be performed, the auditor must identify appropriate criteria. The sources of such criteria are least likely to include A. Benchmarks for the leading firms in the industry. B. Best practices for another industry. C. Historical cost information for the processes examined. D. Government regulations for the industry.

B. Best practices for another industry.

An internal auditor is considering whether the amount of cash is accurately recorded on the financial statements. All of the following are appropriate engagement procedures for the objective except A. Examining bank reconciliations and confirming bank balances. B. Comparing cash receipt lists with the receipts journal and bank deposit slips. C. Verifying cutoff of receipts and disbursements. D. Adding totals of reconciliations and comparing with cash account balances.

B. Comparing cash receipt lists with the receipts journal and bank deposit slips.

A sales department has been giving away expensive items in conjunction with new product sales to stimulate demand. The promotion seems successful, but management believes the cost may be too high and has asked for a review by the internal audit activity. Which of the following procedures would be the least useful to determine the effectiveness of the promotion? A. Comparing product sales during the promotion period with sales during a similar non-promotion period. B. Comparing the unit cost of the products sold before and during the promotion period. C. Performing an analysis of marginal revenue and marginal cost for the promotion period, compared to the period before the promotion. D. Performing a review of the sales department's benchmarks used to determine the success of a promotion.

B. Comparing the unit cost of the products sold before and during the promotion period.

In which of the following arrangements should an internal auditor be most concerned about the lack of an incentive for economy and efficiency? A. Fixed-price contract. B. Cost-plus contract. C. Unit-price contract. D. Source code escrow clause.

B. Cost-plus contract.

A certified internal auditor is the chief audit executive for a large city and is planning the engagement work schedule for the next year. The city has a number of different funds, some that are restricted in use by government grants and some that require compliance reports to the government. One of the programs for which the city has received a grant is job retraining and placement. The grant specifies certain conditions a participant in the program must meet to be eligible for the funding. The chief audit executive plans an engagement to verify that the job retraining program complies with applicable grant provisions. One of the provisions is that the city adopt a budget for the program and subsequently follow procedures to ensure that the budget is adhered to and that only allowable costs are charged to the program. In performing an engagement concerning compliance with this provision, the internal auditors should perform all of the following procedures except A. Determine that the budget was reviewed and approved by supervisory personnel within the city. B. Determine that the budget was reviewed and approved by supervisory personnel within the granting agency. C. Select a sample of expenditures to determine that the expenditures are (1) properly classified as to type, (2) appropriate to the program, and (3) designed to meet the program's objectives. D. Compare actual results with budgeted results and determine the reason for deviations. Determine if such deviations have been approved by appropriate officials.

B. Determine that the budget was reviewed and approved by supervisory personnel within the granting agency.

Internal auditors are increasingly called on to perform audits related to an organization's environmental stewardship. Which of the following does not describe the objectives of a type of environmental audit? A. Determine whether environmental management systems are in place and operating properly to manage future environmental risks. B. Determine whether environmental issues are considered as part of economic decisions. C. Determine whether the organization's current actions are in compliance with existing laws. D. Determine whether the organization is focusing efforts on ensuring that its products are environmentally friendly, and confirm that product and chemical restrictions are met.

B. Determine whether environmental issues are considered as part of economic decisions.

An organization's managerial decision-making model for capital budgeting is based on the net present value of discounted cash flows. The same organization's managerial performance evaluation model is based on annual divisional return on investment. Which of the following is true? A. Divisional managers are likely to maximize the measures in the decision-making model. B. Divisional managers are likely to maximize the measures in the performance evaluation model. C. The manager has an incentive to accept a project with a positive net present value that initially has a negative effect on net income. D. The use of models with different criteria promotes goal congruence.

B. Divisional managers are likely to maximize the measures in the performance evaluation model.

An internal auditor has just completed a survey to become familiar with the organization's payroll operations as part of an unplanned engagement. Which of the following most likely is performed next? A. Assign internal audit personnel. B. Establish initial engagement objectives. C. Write the engagement work program. D. Conduct field work.

B. Establish initial engagement objectives.

Which of the following is an example of a soft control? A. Passwords. B. Ethical culture. C. Segregation of duties. D. Authorization signatures.

B. Ethical culture.

Which of the following is a key to successful total quality management (TQM)? A. Training quality inspectors. B. Focusing intensely on the customer. C. Creating appropriate hierarchies to increase efficiency. D. Establishing a well-defined quality standard, then focusing on meeting it.

B. Focusing intensely on the customer.

Internal audit consulting services A. Involve assuming certain responsibilities for managing risks. B. Include training. C. Exclude investigations and nonaudit roles. D. Add value in ways not necessarily consistent with the definition of internal auditing.

B. Include training.

After testing a client's internal control activities, an auditor discovers a number of significant deficiencies in the operation of a client's internal controls. Under these circumstances, the auditor most likely would A. Issue a disclaimer of opinion about the internal controls as part of the auditor's report. B. Increase the assessment of control risk and increase the extent of substantive tests. C. Issue a qualified opinion of this finding as part of the auditor's report. D. Withdraw from the audit because the internal controls are ineffective.

B. Increase the assessment of control risk and increase the extent of substantive tests.

The budget for an engagement performed by internal auditors A. Should be approximated using the previous audit of the same activity. B. Is adjusted only after approval at a level higher than the engagement supervisor. C. May be increased or decreased by senior management or the board. D. Includes time budgets monitored by the engagement supervisor to control the project.

B. Is adjusted only after approval at a level higher than the engagement supervisor.

According to the International Professional Practices Framework, the internal audit activity is effectively managed when A. Policies on responsibilities of the internal audit activity are included in the organization's operations manual. B. Its individual members conform with the Code of Ethics and the Standards. C. Management oversees the day-to-day operations of the internal audit activity. D. It has the skill set and knowledge to help the organization achieve its objectives.

B. Its individual members conform with the Code of Ethics and the Standards.

An internal auditor performed a formal consulting engagement for XYZ Corporation on June 1, Year 1. When is the earliest time the auditor can perform assurance services for XYZ Corporation and be considered independent and objective? A. January 1, Year 2. B. June 1, Year 2. C. July 1, Year 1. D. June 2, Year 1.

B. June 1, Year 2.

In addressing internal audit resource needs for a complex engagement, the CAE may include all of the following except A. Other employees of the organization. B. Members of the audit committee. C. Specialized consultants. D. External service providers.

B. Members of the audit committee.

Using a balanced scorecard, an organization evaluates managerial performance based on A. A single ultimate measure of operating results, such as residual income. B. Multiple financial and nonfinancial measures. C. Multiple nonfinancial measures only. D. Multiple financial measures only.

B. Multiple financial and nonfinancial measures.

Fact pattern: You are the chief audit executive of a parent organization that has foreign subsidiaries. Independent external audits performed for the parent are not conducted by the same firm that conducts the foreign subsidiary audits. Because the internal audit activity occasionally provides direct assistance to both external firms, you have copies of audit programs and selected working papers produced by each firm. The foreign subsidiary's auditors would like to rely on some of the work performed by the parent organization's audit firm, but they need to review the working papers first. They have asked you for copies of the working papers of the parent organization's audit firm. What is the most appropriate response to the foreign subsidiary's auditors? A. Provide copies of the working papers without notifying the parent's audit firm. B. Notify the parent's auditors of the situation and request that they either provide the working papers or authorize you to do so. C. Provide copies of the working papers and notify the parent's audit firm that you have done so. D. Refuse to provide the working papers under any circumstances.

B. Notify the parent's auditors of the situation and request that they either provide the working papers or authorize you to do so.

When performing procedures to test assertions about purchases, an auditor vouches a sample of entries in the voucher register to the supporting documents. Which relevant assertion would this procedure most likely support? A. Completeness. B. Occurrence. C. Valuation and allocation. D. Classification.

B. Occurrence.

Fact pattern: A purchasing agent acquired items for personal use with the organization's funds. The organization allowed designated employees to purchase a specified amount per day in merchandise under open-ended contracts. Supervisory approval of the purchases was required, but that information was not communicated to the vendor. Instead of reviewing and authorizing each purchase order, supervisors routinely signed the authorization sheet at the end of the month without reviewing any of the supporting documentation. Because purchases of this nature were not subject to normal receiving policies, the dishonest employee picked up the supplies at the vendor's warehouse. All purchases were for items routinely ordered by the organization. During the past year, the employee amassed enough merchandise to start a printing and photography business. Which of the following engagement procedures, performed by the internal auditor, is most likely to detect this fraud? A. Tracing selected canceled checks to the cash payments journal and to the related vendors' invoices. B. Performing a trend analysis of printing supplies expenses for a 2-year period. C. Tracing prices and quantities on selected vendors' invoices to the related purchase orders. D. Recomputing the clerical accuracy of selected vendors' invoices, including discounts and sales taxes.

B. Performing a trend analysis of printing supplies expenses for a 2-year period.

The chief audit executive (CAE) is best defined as the A. Inspector general. B. Person responsible for the internal audit function C. Outside provider of internal audit services. D. Person responsible for overseeing the contract with the outside provider of internal audit services.

B. Person responsible for the internal audit function.

Data-gathering activities such as interviewing operating personnel, identifying standards to be used to evaluate performance, and assessing risks inherent in a department's operations are typically performed in which phase of an audit engagement? A. Field work. B. Preliminary survey. C. Engagement program development. D. Examination and evaluation of evidence.

B. Preliminary survey.

Which type of engagement focuses on operations and how effectively and efficiently the organizational units affected will cooperate? A. Program-results engagement. B. Process engagement. C. Privacy engagement. D. Compliance engagement.

B. Process engagement.

Which of the following is an example of business process reengineering? A. Adding a new machine to the existing production line to speed up production. B. Redesigning the production line to speed up production. C. Repairing a machine on the process line to speed up production. D. Updating the computer systems involved on the production line to speed up production

B. Redesigning the production line to speed up production.

During an engagement to evaluate travel expenses, the accounting supervisor tells the internal auditor that each expense report is reviewed and approved before costs are reimbursed to the traveler. Which of the following is the best course of action for the internal auditor to take? A. Request the supervisor to put the statement in writing. B. Review a sample of expense reports for proper approval. C. Conserve engagement resources by accepting the statement and redirect work into another area. D. Corroborate this information with the controller.

B. Review a sample of expense reports for proper approval.

Which of the following procedures is the most valuable in an engagement involving the traffic department operations of a large manufacturer? A. Obtain written confirmation from the regulatory agencies that all carriers used are properly licensed and bonded. B. Review procedures for selection of routes and carriers. C. Trace selected items from the weekly demurrage (car detention charge) report to supporting documentation. D. Verify that all bills of lading are prenumbered.

B. Review procedures for selection of routes and carriers.

Which of the following procedures would provide the best evidence of the effectiveness of a credit-granting function? A. Observe the process. B. Review the trend in receivables write-offs. C. Ask the credit manager about the effectiveness of the function. D. Check for evidence of credit approval on a sample of customer orders.

B. Review the trend in receivables write-offs.

Which of the following is an appropriate responsibility of the board? A. Performing a review of the procurement function of the organization. B. Reviewing the internal audit activity's engagement work schedule submitted by the chief audit executive. C. Reviewing the engagement records of the public accounting firm to determine the firm's competence. D. Recommending the assignment of specific internal audit staff members for specific engagements.

B. Reviewing the internal audit activity's engagement work schedule submitted by the chief audit executive.

In confirming with an outside agent, such as a financial institution, that the agent is holding investment securities in the client's name, an auditor most likely gathers evidence in support of relevant financial statement assertions about existence or occurrence and A. Valuation and allocation. B. Rights and obligations. C. Completeness. D. Classification and understandability.

B. Rights and obligations.

While planning an engagement, an internal auditor establishes engagement objectives to describe what is to be accomplished. Which of the following is a key issue to consider in developing engagement objectives? A. The qualifications of the internal auditing staff selected for the engagement. B. Risks associated with the activities to be reviewed. C. Recommendations of the engagement client's employees. D. The recipients of the final engagement communication.

B. Risks associated with the activities to be reviewed.

Which of the following is not an engagement objective related to the purchasing function? A. Determine whether purchases eligible for competitive bids are properly reviewed and authorized. B. Run background checks on unauthorized vendors. C. Determine whether receiving reports are independently verified. D. Determine whether goods received are properly reflected in purchasing records.

B. Run background checks on unauthorized vendors.

Which of the following activities is outside the scope of internal auditing? A. Evaluating risk exposures regarding compliance with policies, procedures, and contracts. B. Safeguarding of assets. C. Evaluating risk exposures regarding compliance with laws and regulations. D. Ascertaining the extent to which management has established criteria to determine whether objectives have been accomplished.

B. Safeguarding of assets.

Fact pattern: An internal auditor is performing an operational engagement at a division and observes that an unusually large quantity of goods is on hand in the shipping and materials rework areas. The items are labeled as re-ship items. Upon inquiry the internal auditor is told that these are goods that have been returned by customers and have either been repaired and shipped back to the original customer or repaired and shipped out as new products because they are fully warranted. The internal auditor has not yet performed any detailed engagement work. Based on the information given, the most appropriate action for the internal auditor to take is to A. Report the items to divisional management and ask for management's explanation before determining whether to include the observations in an engagement communication. B. Take a sample of the items on hand and compare the sample items with the underlying documents, such as receiving reports and sales orders, to determine how the goods were handled. C. Report the observations but do not perform any additional work without the approval of the chief audit executive because such work is clearly a scope expansion. D. Take an inventory of the goods on hand so the monetary amount can be included in the engagement communications along with the explanation of the problem.

B. Take a sample of the items on hand and compare the sample items with the underlying documents, such as receiving reports and sales orders, to determine how the goods were handled.

Internal audit resources should be appropriate, sufficient, and effectively deployed. Consequently, A. Resource planning should be limited to expected activities. B. The chief audit executive should perform a periodic skills assessment. C. Only members of the internal audit staff should perform internal audit activities. D. The chief audit executive ultimately must ensure the adequacy of resources.

B. The chief audit executive should perform a periodic skills assessment.

The internal audit activity's audit plan is based on all of the following except A. The audit universe. B. The cost of the engagement. C. Input from senior management and the board. D. Assessed risk and exposures.

B. The cost of the engagement.

The chief audit executive is preparing the audit work schedule for the next budget year and has limited resources. In deciding whether to schedule the purchasing or the personnel department for an engagement, which of the following is the least important factor? A. Major changes in operations have occurred in one of the departments. B. The internal audit staff has recently added an individual with expertise in one of the areas. C. More opportunities to achieve operating benefits are available in one of the departments than in the other. D. Updated assessed risk is significantly greater in one department than the other.

B. The internal audit staff has recently added an individual with expertise in one of the areas.

Which of the following possible engagement objectives would lead to a test of the efficiency of an organization's use of labor resources? A. To determine that all employees are paid in accordance with union wages. B. To determine that employees are assigned to work situations equivalent to their training and skill level. C. To determine that the quality of performance by labor meets organizational standards. D. To determine that only authorized employees are paid.

B. To determine that employees are assigned to work situations equivalent to their training and skill level.

Which of the following is an appropriate statement of an engagement objective? A. To observe the physical inventory count. B. To determine whether inventory stocks are sufficient to meet projected sales. C. To search for the existence of obsolete inventory by computing inventory turnover by product line. D. To include information about stockouts in the final engagement communication.

B. To determine whether inventory stocks are sufficient to meet projected sales.

What is the purpose of establishing an internal audit plan? A. To update the audit universe. B. To ensure adequate coverage of areas with the greatest exposure to risks. C. To identify areas of audits with lower risks. D. To identify, document, and analyze the means by which management mitigates the risks.

B. To ensure adequate coverage of areas with the greatest exposure to risks.

In planning an engagement, the internal auditor establishes objectives to address the risk associated with the activity. Risk is the A. Possibility that the balance or class of transactions and related assertions contains misstatements that could be material to the financial statements. B. Uncertainty of the occurrence of an event that could affect the achievement of objectives. C. Failure to adhere to organizational policies, plans, and procedures or to comply with relevant laws and regulations. D. Failure to accomplish established objectives and goals for operations or programs.

B. Uncertainty of the occurrence of an event that could affect the achievement of objectives.

Gerald Fitz, CAE, believes that the internal controls over cash disbursements need major revisions. He discussed this matter with senior management and was alarmed at their acceptance of this serious risk. The CAE should A. Report the matter to the board immediately. B. Understand management's basis for accepting the risk. C. Determine whether management has the authority to accept the risk. D. Further attempt to resolve the disagreement.

B. Understand management's basis for accepting the risk.

An internal auditor performs a test to determine whether all merchandise for which the organization was billed was received. The population for this test consists of all A. Merchandise received. B. Vendors' invoices. C. Canceled checks. D. Receiving reports.

B. Vendors' invoices.

Levels of production stoppages over the past year at a large laminating business were abnormally high due to machine malfunctions. Would it be appropriate for the internal auditing function to develop a survey examining attitudes toward line operations, rotation of work zones, training, maintenance schedule, etc., for the machine operators to complete? A. Yes, the survey is reliable without corroboration. B. Yes, the examined areas are relevant to the malfunctions. C. No, the examined areas are irrelevant to the malfunctions. D. No, the survey is inappropriate without corroboration.

B. Yes, the examined areas are relevant to the malfunctions.

Briar Co. signed a government construction contract providing for a formula price of actual cost plus 10%. In addition, Briar was to receive one-half of any savings resulting from the formula price's being less than the target price of $2.2 million. Briar's actual costs incurred were $1,920,000. How much should Briar receive from the contract? A. $2,060,000 B. $2,112,000 C. $2,156,000 D. $2,200,000

C. $2,156,000

An organization should use due care not to delegate substantial discretionary authority to individuals the organization knows have a propensity to engage in illegal activities. Which of the following are steps an organization can take to ensure that such individuals are detected? 1. Screening of applicants for employment at all levels for evidence of past wrongdoing, especially past criminal convictions within the company's industry. 2. Asking professionals about any history of discipline in front of licensing boards. 3. Performing background checks without permission on employees' or applicants' credit reports to ensure that they are financially sound and are unlikely to commit theft or fraud. A. 1 only. B. 3 only. C. 1 and 2 only. D. 1, 2, and 3.

C. 1 and 2 only.

A chief audit executive (CAE) uses a risk assessment model to establish the annual audit plan. Which of the following would be an appropriate action by the CAE? 1. Maintain ongoing dialogue with management and the audit committee 2. Ensure that the schedule of audit priorities remains unchanged 3. Employ only quantitative methods to determine risk weightings 4. Revise the risk assessment and audit priorities as warranted A. 3 only. B. 1 and 2 only. C. 1 and 4 only. D. 3 and 4 only.

C. 1 and 4 only.

Risk modeling in a consulting service can be accomplished by 1. Ranking the engagement's potential to improve management of risks 2. Ranking the engagement's potential to add value 3. Ranking the engagement's potential to improve the organization's operations A. 1 and 2. B. 1 and 3. C. 1, 2, and 3. D. 3 only.

C. 1, 2, and 3.

Which of the following potentially is (are) subject to the internal auditors' evaluations? 1. The human resources function. 2. The purchasing process. 3. The manufacturing and production database system. A. 1 only. B. 2 only. C. 1, 2, and 3. D. None of the answers are correct.

C. 1, 2, and 3.

In deciding whether to accept a consulting engagement, the Standards require the CAE to consider the engagement's potential to 1. Add value 2. Improve management of risks 3. Develop internal audit competencies 4. Improve the organization's operations A. 1 only. B. 1 and 2 only. C. 1, 2, and 4 only. D. 1, 2, 3, and 4.

C. 1, 2, and 4 only.

Before internal auditors begin to offer consulting services to an organization, a number of things need to happen within the organization. What is the order in which the following items should be performed? 1. The internal audit charter is amended to include authority and responsibilities for consulting activities. 2. The CAE confirms that the board understands and approves the concept of providing consulting services. 3. The internal audit activity develops appropriate policies and procedures for conducting such engagements. A. 1, 2, 3. B. 2, 3, 1. C. 2, 1, 3. D. 3, 2, 1.

C. 2, 1, 3.

An internal auditor concludes that the results of a consulting engagement should be communicated beyond those who received or requested the services. The auditor follows a series of steps until satisfied with the resolution. In what order will the auditor perform the following steps? 1. Attempt to convince those receiving or requesting the service to expand voluntarily the communication to the appropriate parties. 2. Determine what guidance is provided in the organization's code of conduct, code of ethics, and other relevant policies, administrative directives, or procedures. 3. Determine what direction is provided in the agreement concerning the consulting engagement and related communications. A. 2, 1, 3. B. 1, 2, 3. C. 3, 1, 2. D. 1, 3, 2.

C. 3, 1, 2.

The key factor in the success of an internal audit activity's human resources program is A. An informal program for developing and counseling staff. B. A compensation plan based on years of experience. C. A well-developed set of selection criteria. D. A program for recognizing the special interests of individual staff members.

C. A well-developed set of selection criteria.

Which of the following engagement objectives will be accomplished by tracing a sample of accounts receivable debit entries to customer invoices and related shipping documents? A. Sales are properly recorded. B. Sales are billed at the correct prices. C. Accounts receivable represent valid sales. D. Customer credit is approved.

C. Accounts receivable represent valid sales.

Which of the following represents an external risk factor? A. The organization's CEO unexpectedly became ill and had to resign. The chairman of the board of directors stepped into the vacant role until a new CEO could be found. B. Constant repairs to outdated equipment used in the manufacturing process cost three times more than the amount budgeted. C. Additional safety regulations enacted by the government have caused a strain on the organization's resources. D. Weak controls over cash accounts have resulted in employee theft.

C. Additional safety regulations enacted by the government have caused a strain on the organization's resources.

As part of a manufacturing company's environmental, health, and safety (EHS) self-inspection program, inspections are conducted by a member of the EHS staff and the operational manager for a given work area or building. If a deficiency cannot be immediately corrected, the EHS staff member enters it into a tracking database that is accessible to all departments via a local area network. The EHS manager uses the database to provide senior management with quarterly activity reports regarding corrective action. During review of the self-inspection program, an auditor notes that the operational manager enters the closure information and affirms that corrective action is complete. What change in the control system would compensate for this potential conflict of interest? A. No additional control is needed because the quarterly report is reviewed by senior management, providing adequate oversight in this situation. B. No additional control is needed because those implementing a corrective action are in the best position to evaluate the adequacy and completion of that action. C. After closure is entered into the system, review by the EHS staff member of the original inspection team should be required in order to verify closure. D. The EHS department secretary should be responsible for entering all information into the tracking system based on memos from the operational manager.

C. After closure is entered into the system, review by the EHS staff member of the original inspection team should be required in order to verify closure.

Employees have the most confidence in a hotline monitored by which of the following? A. An expert from the legal department, backed by a nonretaliation policy. B. An in-house representative, backed by a retaliation policy. C. An on-site ombudsperson, backed by a nonretaliation policy. D. An off-site attorney who can better protect attorney-client privilege.

C. An on-site ombudsperson, backed by a nonretaliation policy.

The manager of a production line has the authority to order and receive replacement parts for all machinery that requires periodic maintenance. The internal auditor received an anonymous tip that the manager ordered substantially more parts than were necessary from a family member in the parts supply business. The unneeded parts were never delivered. Instead, the manager processed receiving documents and charged the parts to machinery maintenance accounts. The payments for the undelivered parts were sent to the supplier, and the money was divided between the manager and the family member. Which of the following tests would best assist the auditor in deciding whether to investigate this anonymous tip further? A. Comparison of the current quarter's maintenance expense with prior-period activity. B. Physical inventory testing of replacement parts for existence and valuation. C. Analysis of repair parts charged to maintenance to review the reasonableness of the number of items replaced. D. Review of a test sample of parts invoices for proper authorization and receipt.

C. Analysis of repair parts charged to maintenance to review the reasonableness of the number of items replaced.

A performance audit engagement typically involves A. Review of financial statement information, including the appropriateness of various accounting treatments. B. Tests of compliance with policies, procedures, laws, and regulations. C. Appraisal of the business and control environment and comparison against established criteria. D. Evaluation of organizational and departmental structures, including assessments of process flows.

C. Appraisal of the business and control environment and comparison against established criteria.

The management of a company would do which of the following to compare and contrast its financial information to published information reflecting optimal amounts? A. Budget. B. Forecast. C. Benchmark. D. Utilize best practices.

C. Benchmark.

What is the process by which products and services of a business entity are measured and evaluated relative to the best possible levels of performance? A. Measuring the performance gap. B. Standard measurement. C. Benchmarking. D. Variance management.

C. Benchmarking.

Which of the following is true about the principle of value proposition to an organization? A. The internal audit function does not add value to an organization. B. Only the consulting activities of the internal audit function provide value. C. Both the assurance and consulting activities add value to the organization. D. Only the assurance activities of the internal audit function add value to the organization.

C. Both the assurance and consulting activities add value to the organization.

An internal auditor conducts a preliminary survey and identifies a number of significant engagement issues and reasons for pursuing them in more depth. The engagement client informally communicates concurrence with the preliminary survey results and asks that the internal auditor not report on the areas of significant concern until the client has an opportunity to respond to the problem areas. Which of the following engagement responses is not appropriate? A. Keep the engagement on schedule and discuss with management the need for completing the engagement on a timely basis. B. Consider the risk involved in the areas involved, and, if the risk is high, proceed with the engagement. C. Consider the engagement to be terminated with no communication of results needed because the engagement client has already agreed to take constructive action. D. Work with the engagement client to keep the engagement on schedule and address the significant issues in more depth, as well as the client's responses, during the course of the engagement.

C. Consider the engagement to be terminated with no communication of results needed because the engagement client has already agreed to take constructive action.

As a particular engagement is being planned in a high-risk area, the chief audit executive determines that the available staff does not have the requisite skills to perform the assignment. The best course of action consistent with engagement planning principles is to A. Not perform the engagement because the requisite skills are not available. B. Use the engagement as a training opportunity and let the internal auditors learn as the engagement is performed. C. Consider using external resources to supplement the needed knowledge, skills, and other competencies and complete the assignment. D. Perform the engagement but limit the scope in light of the skill deficiency.

C. Consider using external resources to supplement the needed knowledge, skills, and other competencies and complete the assignment.

To improve their efficiency, internal auditors may rely upon the work of external auditors if it is A. Performed after the internal auditing work. B. Primarily concerned with operational objectives and activities. C. Coordinated with internal auditing work. D. Conducted in accordance with the Code of Ethics.

C. Coordinated with internal auditing work.

An auditor is conducting a performance audit to provide assurance on an organization's balanced scorecard. The organization's main objective is to increase market share by 7% in the coming year. Management diverted 5% of the operating budget from the customer service department to the research and development department to increase product innovation. Management had predicted that increased product innovation would increase market share. However, market share did not increase substantially in the first quarter. Which measure should the auditor review as a result of the failure to increase market share? A. Product innovation. B. Market share. C. Customer satisfaction. D. Employee development.

C. Customer satisfaction.

An annual summary report of completed engagement work submitted to senior management and the board by the chief audit executive should A. Discuss the administrative condition of the internal audit activity. B. Inform management of the scope of proposed work for the following year. C. Describe the extent to which the internal audit activity has completed its approved audit plan. D. Emphasize the number of deficiency observations discovered by the internal auditors.

C. Describe the extent to which the internal audit activity has completed its approved audit plan.

Which of the following types of risk increases when an auditor performs substantive analytical audit procedures for financial statement accounts at an interim date? A. Inherent. B. Control. C. Detection. D. Sampling.

C. Detection.

Which of the following types of risks most likely would increase if accounts receivable are confirmed 3 months before year end? A. Inherent. B. Control. C. Detection. D. Business.

C. Detection.

An external consultant is developing methods for the management of a city's capital facilities. An appropriate scope of an engagement to evaluate the consultant's product is to A. Review the consultant's contract to determine its propriety. B. Establish the parameters of the value of the items being managed and controlled. C. Determine the adequacy of the risk management and control systems for the management of capital facilities. D. Review the handling of idle equipment.

C. Determine the adequacy of the risk management and control systems for the management of capital facilities.

The internal auditor has gained an understanding of the design of an engagement client's internal controls. The most appropriate next step is to A. Test controls to determine whether they are functioning as designed. B. Halt the engagement and issue a report about inadequate controls. C. Draw preliminary conclusions about internal control. D. Contact the engagement client's direct supervisor to recommend that the head of the department or function under audit is transferred or terminated.

C. Draw preliminary conclusions about internal control.

The internal audit activity has recently experienced the departure of two internal auditors who cannot be immediately replaced due to budget constraints. Which of the following is the least desirable option for efficiently completing future engagements, given this reduction in resources? A. Using self-assessment questionnaires to address audit objectives. B. Employing information technology in audit planning, sampling, and documentation. C. Eliminating consulting engagements from the engagement work schedule. D. Filling vacancies with personnel from operating departments that are not being audited.

C. Eliminating consulting engagements from the engagement work schedule.

An internal auditing supervisor reviewed the system of controls and the organizational objective of the purchasing department. What facet of engagement planning was the supervisor developing? A. Internal auditing policy manual. B. Engagement work schedule. C. Engagement work program. D. Internal auditing budget.

C. Engagement work program.

The IIA's Three Lines Model states that the roles of an organization's governing body most likely include A. Assisting with risk management. B. Delivering products to clients. C. Ensuring that organizational objectives align with stakeholders' interests. D. Providing assurance and advice that instills confidence and clarity.

C. Ensuring that organizational objectives align with stakeholders' interests.

For review of an accounting department's bank reconciliation unit, which of the following is an appropriate engagement work program step for the review of canceled checks for authorized signatures? A. Comparing the check date with the first cancellation date. B. Determining that all checks are to be signed by individuals authorized by the board. C. Examining a representative sample of signed checks and determining that the signatures are authorized in the organizational signature book. D. Completing the tests of controls over check signatures in 4 hours.

C. Examining a representative sample of signed checks and determining that the signatures are authorized in the organizational signature book.

The audit committee strengthens the control processes of an organization by A. Assigning the internal audit activity responsibility for interaction with governmental agencies. B. Using the chief audit executive as a major resource in selecting the external auditors. C. Following up on recommendations made by the chief audit executive. D. Approving internal audit activity policies.

C. Following up on recommendations made by the chief audit executive.

An internal auditor's observation of physical inventories at the main plant at year end provides direct evidence to support which of the following objectives? A. Accuracy of the priced-out inventory. B. Evaluation of lower of cost or market test. C. Identification of obsolete or damaged merchandise to evaluate allowance (reserve) for obsolescence. D. Determination of goods on consignment at another location.

C. Identification of obsolete or damaged merchandise to evaluate allowance (reserve) for obsolescence.

A service company is currently experiencing a significant downsizing and process reengineering. Its board of directors has redefined the business goals and established initiatives using in-house developed technology to meet these goals. As a result, a more decentralized approach has been adopted to run the business functions by empowering the business branch managers to make decisions and perform functions traditionally done at a higher level. The internal auditing staff is made up of the director, two managers, and five staff auditors, all with financial background. In the past, the primary focus of successful audit activities has been the service branches and the six regional division headquarters that support the branches. These division headquarters are the primary targets for possible elimination. The support functions, such as human resources, accounting, and purchasing, will be brought into the national headquarters, and technology will be enhanced to enable and augment these operations. Assuming that total available resources remain the same, what activities should the internal audit activity perform to best serve the organization? A. Decrease engagement time in systems development. B. Increase engagement time in service branches. C. Increase engagement time in functions being centralized. D. Continue the allocation of engagement time as before.

C. Increase engagement time in functions being centralized.

Which statement about consulting engagements is true? A. Documentation requirements applicable to assurance engagements apply to consulting engagements. B. The internal audit activity may assume management responsibility to the extent agreed upon with the client. C. Internal auditors keep senior management and the board informed about how audit resources are being deployed. D. Work programs for formal consulting engagements address policies and issues related to ownership of consulting engagement records to protect the organization and avoid any potential misunderstandings.

C. Internal auditors keep senior management and the board informed about how audit resources are being deployed.

The reliability and integrity of all critical information of an organization, regardless of the media in which the information is stored, is the responsibility of A. Shareholders. B. IT department. C. Management. D. All employees.

C. Management.

Internal auditors need to consider protection of personally identifiable information obtained during an audit. Applicable laws most likely A. Do not establish requirements for an organization to implement privacy controls. B. Permit personal information to be used for any purpose if disclosure of a purpose was made at collection. C. May prohibit recording personal information in engagement records in some cases. D. Require personal information to be encrypted when recorded and stored in digital form.

C. May prohibit recording personal information in engagement records in some cases.

The internal audit activity of a large organization has established its operating plan and budget for the coming year. The operating plan is restricted to the following categories: a prioritized listing of all engagements, staffing, a detailed expense budget, and the commencement date of each engagement. Which of the following best describes the major deficiency of this operating plan? A. Requests by management for special projects are not considered. B. Opportunities to achieve operating benefits are ignored. C. Measurability criteria and targeted dates of completion are not provided. D. Knowledge, skills, and other competencies required to perform work are ignored.

C. Measurability criteria and targeted dates of completion are not provided.

The chief audit executive was reviewing recent reports that had recommended additional engagements because of risk exposures to the organization. Which of the following represents the greatest risk and should be the next assignment? A. Three prenumbered receiving reports were missing. B. There were several purchase orders issued without purchase requisitions. C. Payment had been made for routine inventory items without a purchase order or receiving report. D. Several times cash receipts had been held over an extra day before depositing.

C. Payment had been made for routine inventory items without a purchase order or receiving report.

The internal auditors' ultimate responsibility for information security includes A. Identifying technical aspects, risks, processes, and transactions to be examined. B. Determining the scope and degree of testing to achieve engagement objectives. C. Periodically assessing information security practices. D. Documenting engagement procedures.

C. Periodically assessing information security practices.

All of the following would be part of a factory's control system to prevent release of wastewater that does not meet discharge standards except A. Performing chemical analysis of the water, prior to discharge, for components specified in the permit. B. Specifying (by policy, training, and advisory signs) which substances may be disposed of via sinks and floor drains within the factory. C. Periodically flushing sinks and floor drains with a large volume of clean water to ensure pollutants are sufficiently diluted. D. Establishing a preventive maintenance program for the factory's pretreatment system.

C. Periodically flushing sinks and floor drains with a large volume of clean water to ensure pollutants are sufficiently diluted.

A work program for a comprehensive assurance engagement to evaluate a purchasing function should include A. Procedures arranged by relative priority based upon perceived risk. B. A statement of the engagement objectives for the operation under review with agreement by the engagement client. C. Procedures to accomplish engagement objectives. D. A focus on risks affecting the financial statements as opposed to controls.

C. Procedures to accomplish engagement objectives.

Which of the following is an appropriate objective in an engagement to review a personnel department? Determining whether A. Hourly employees are being paid only for hours actually worked as indicated by time cards or similar reports. B. An equitable training program exists that provides all employees with approximately the same amount of training each year. C. Reference checks of prospective employees are being performed. D. Recruitment is being delegated to the various departments that have personnel needs.

C. Reference checks of prospective employees are being performed.

An audit found that the cost of some material installed on capital projects had been transferred to the inventory account because the capital budget had been exceeded. Which of the following would be an appropriate technique for the internal audit activity to use to monitor this situation? A. Identify variances between amounts capitalized each month and the capital budget. B. Analyze a sample of capital transactions each quarter to detect instances in which installed material was transferred to inventory. C. Review all journal entries that transferred costs from capital projects accounts to inventory accounts. D. Compare inventory receipts with debits to the inventory account and investigate discrepancies.

C. Review all journal entries that transferred costs from capital projects accounts to inventory accounts.

Fact pattern: You are an internal auditing supervisor who is reviewing the working papers of a staff internal auditor's overall examination of the firm's sales function. The pages are not numbered or cross-referenced. Furthermore, the working papers were dropped and reassembled at random before they were brought to you. You decide to put the working papers in the proper order according to the Standards. The first stage of this activity is to identify each page as a part of the preliminary survey, the review of the adequacy of control processes, the review for effectiveness of control processes, or the review of results. The first page the supervisor selects documents a test of controls performed during the course of the engagement. This page belongs with which activity? A. Preliminary survey. B. Review for adequacy of control processes. C. Review for effectiveness of control processes. D. Review of results.

C. Review for effectiveness of control processes.

An auditor is scheduled to audit payroll controls for an organization that has recently outsourced its information processing to an external service provider (ESP). The ESP's external auditor has issued reports pertaining to the ESP's controls and made it readily available to the internal auditor. What action should the auditor take, considering the outsourcing decision? A. Review only the ESP's external auditor. B. Review only the organization's controls over data sent to and received from the ESP. C. Review the control reports and ensure that the ESP's external auditor is credible and reliable. D. Cancel the engagement because the processing is being performed outside of the organization.

C. Review the control reports and ensure that the ESP's external auditor is credible and reliable.

A chief audit executive is reviewing the following enterprise-wide risk map: I M P A C T LIKELIHOOD Remote Possible Likely Critical Risk A Risk B Major Risk D Minor Risk C Which of the following is the correct prioritization of risks, considering limited resources in the internal audit activity? A. Risk B, Risk C, Risk A, Risk D. B. Risk A, Risk B, Risk C, Risk D. C. Risk D, Risk B, Risk C, Risk A. D. Risk B, Risk C, Risk D, Risk A.

C. Risk D, Risk B, Risk C, Risk A.

Which of the following is least likely to be placed on the agenda for discussion at a pre-engagement meeting? A. Objectives and scope of the engagement. B. Client personnel needed. C. Sampling plan and key criteria. D. Expected starting and completion dates.

C. Sampling plan and key criteria.

An engagement objective is to determine if a company's accounts payable contain all outstanding liabilities. Which of the following audit procedures would not be relevant for this objective? A. Examine supporting documentation of subsequent (after-period) cash disbursements and verify period of liability. B. Send confirmations, including zero-balance accounts, to vendors with whom the company normally does business. C. Select a sample of accounts payable from the accounts payable listing and verify the supporting receiving reports, purchase orders, and invoices. D. Trace receiving reports issued before the period end to the related vendor invoices and accounts payable listing.

C. Select a sample of accounts payable from the accounts payable listing and verify the supporting receiving reports, purchase orders, and invoices.

When counting cash on hand, the internal auditor must exercise control over all cash and other negotiable assets to prevent A. Theft. B. Irregular endorsement. C. Substitution. D. Deposits in transit.

C. Substitution.

Privacy of space is best defined as freedom from A. Invasion of physical privacy. B. Monitoring of communications. C. Surveillance. D. Disclosure of personal information by others.

C. Surveillance.

Which of the following statements about the chief audit executive's responsibilities for internal audit resources is most accurate? A. The CAE is responsible for ensuring that audit coverage is based on the skills of the internal audit activity. B. The CAE is responsible for presenting a detailed summary of audit resources to management. C. The CAE is responsible for the effective deployment of resources to achieve the approved audit plan. D. The CAE is responsible for administering the organization's compensation program.

C. The CAE is responsible for the effective deployment of resources to achieve the approved audit plan.

Which of the following is true about the interaction of the internal audit function and the environmental audit function? A. If the environmental audit function reports to someone other than the CAE, the CAE should not offer to review the audit plan since (s)he was not consulted to do so. B. It is not advantageous for the internal audit function to conduct environmental audits since it is too busy with its current responsibilities. C. The CAE should evaluate whether the environmental auditors are conforming to recognized professional auditing standards and a recognized code of ethics. D. The CAE should not evaluate the organizational placement and independence of the environmental audit function since the internal function has no control over a separate environmental audit function.

C. The CAE should evaluate whether the environmental auditors are conforming to recognized professional auditing standards and a recognized code of ethics.

Which of the following statements, if true, would justify a chief audit executive's decision not to report certain control concerns regarding derivatives trading in a report to the audit committee? A. Management plans to initiate corrective action. B. The board has a separate committee to make recommendations on trading issues. C. The amounts of trading and the potential risks associated with the derivatives trading are not material to the overall organization. D. Derivatives are complex, and the auditor should rely on management's analysis of the extent of the problem.

C. The amounts of trading and the potential risks associated with the derivatives trading are not material to the overall organization.

Which of the following statements is true regarding coordination of internal and external auditing efforts? A. The chief audit executive should not give information about illegal acts to an external auditor because external auditors may be required to report the matter to the board or regulatory agencies. B. Ownership and the confidentiality of the external auditor's working papers prohibit their review by internal auditors. C. The chief audit executive should determine that appropriate follow-up and corrective action was taken by management when required regarding matters discussed in the external auditor's management letter. D. If internal auditors provide assistance to the external auditors in connection with the annual audit, such assistance is not subject to the Standards.

C. The chief audit executive should determine that appropriate follow-up and corrective action was taken by management when required regarding matters discussed in the external auditor's management letter.

Which of the following statements regarding the external auditor is true? A. Disputes between the external auditor and management are resolved through an arbitrator. B. Review of the external auditor's internal control and audit reports during each engagement is done by a different accounting firm. C. The external auditor's work is overseen and reviewed by the audit committee. D. Negotiation of the external auditor's fee is the responsibility of the corporate officers.

C. The external auditor's work is overseen and reviewed by the audit committee.

After the chief audit executive receives approval from the board to offer consulting services, what should be done? A. The CAE should begin performing consulting services. B. The CAE should get approval from the internal auditors. C. The internal audit charter should be amended. D. The board should develop appropriate policies and procedures for conducting such engagements.

C. The internal audit charter should be amended.

Which of the following factors should an internal auditor consider when planning an audit of an activity? A. The objectives of the activity, the number of employees involved, and the control system. B. The qualifications of management, the significant risks, and the control system. C. The objectives of the activity, the significant risks, and the control system. D. The number of employees involved, the control system, and the recommendations of external auditors.

C. The objectives of the activity, the significant risks, and the control system.

Which of the following is a valid reason for an internal auditing engagement involving a payroll department to receive priority over a purchasing department engagement? A. The director of the payroll department requested that the payroll department engagement be performed first. B. The purchasing department engagement will require more time to perform. C. The payroll department's relative risk and exposure are greater. D. The purchasing department recently restructured its major operations.

C. The payroll department's relative risk and exposure are greater.

Consulting services performed by internal auditors most likely benefit the organization because A. They need not be defined in the internal audit charter. B. The value proposition applies only to assurance services. C. They are intended to improve the organization's controls. D. The constraints of The IIA Code of Ethics do not apply.

C. They are intended to improve the organization's controls.

The internal audit activity (IAA) is effectively managed when A. Senior management creates its operating budget. B. The organization's human resources department hires the IAA's associates. C. Trends and emerging issues are considered. D. The board establishes policies and procedures for the IAA.

C. Trends and emerging issues are considered.

Which technique is most appropriate for testing the quality of the preliminary survey of payment vouchers described in an internal control questionnaire? A. Analysis. B. Evaluation. C. Verification. D. Observation.

C. Verification.

Which of the following are responsibilities of the chief audit executive (CAE)? 1. Coordinating activities with other providers of assurance and consulting services. 2. Understanding the work of external auditors. 3. Providing sufficient information to the external auditors to permit them to understand the internal auditors' work. A. 1 and 2 only. B. 2 and 3 only. C. 1 and 3 only. D. 1, 2, and 3.

D. 1, 2, and 3.

Which of the following is the proper way for an internal auditor to resolve conflict? 1. By the guidelines set out in the organization's code of conduct 2. By the guidelines set out in The IIA's Code of Ethics 3. The procedures designated by the CAE A. 1 and 2. B. 3 only. C. 1, 2, and 3. D. 2 only.

D. 2 only.

Which one of the following is not a core principle of total quality management (TQM)? A. A focus on customers and stakeholders. B. Participation and teamwork by everyone in the organization. C. A process focus supported by continuous improvement and learning. D. A focus on technological breakthroughs.

D. A focus on technological breakthroughs.

Which of the following best describes a preliminary survey? A. A standardized questionnaire used to obtain an understanding of management objectives. B. A statistical sample of key employee attitudes, skills, and knowledge. C. A "walk-through" of the financial control system to identify risks and the controls that can address those risks. D. A process used to become familiar with activities and risks to identify areas for engagement emphasis.

D. A process used to become familiar with activities and risks to identify areas for engagement emphasis.

Which of the following criteria would be most useful to a sales department manager in evaluating the performance of the manager's customer-service group? A. The customer is always right. B. Customer complaints should be processed promptly. C. Employees should maintain a positive attitude when dealing with customers. D. All customer inquiries should be answered within 7 days of receipt.

D. All customer inquiries should be answered within 7 days of receipt.

Internal audit engagements require budgeting and staff scheduling. Which of the following is true? A. Budgets are developed for the most important engagements. B. Use of actual engagements as training opportunities is discouraged. C. Time budgets ordinarily are prepared based on supervision requirements. D. All engagements should be under budgetary control.

D. All engagements should be under budgetary control.

Control self-assessment (CSA) is a method for examining and evaluating the organization's system of control, which includes A. Risk analysis. B. Self-assessment approaches. C. Traditional internal auditing concepts. D. All of the answers are correct.

D. All of the answers are correct.

During consulting engagements, internal auditors A. May gain knowledge of risks that should be included in evaluating organizational risk management. B. Are responsible for significant risks not consistent with the objectives. C. May gain knowledge of controls that should be included in evaluating organizational controls. D. All of the answers are correct.

D. All of the answers are correct.

Substantial risk exposures or material control weaknesses discovered during a formal consulting engagement are brought to the attention of management. In some situations, the internal auditor's concerns also are communicated to A. Executive management. B. Audit committee. C. Board of directors. D. All of the answers are correct.

D. All of the answers are correct.

Use of external service providers with expertise in healthcare benefits is appropriate when the internal audit activity is A. Evaluating the organization's estimate of its liability for postretirement benefits, which include healthcare benefits. B. Comparing the cost of the organization's healthcare program with other programs offered in the industry. C. Training its staff to conduct an audit of healthcare costs in a major division of the organization. D. All of the answers are correct.

D. All of the answers are correct.

Developing engagement observations, conclusions, and recommendations involves comparing the condition with the relevant standard or criterion. Which of the following choices best represents an appropriate standard or criterion to support engagement observations, conclusions, and recommendations? A. A quality standard operating procedure (number and date) for the department. B. An internal accounting control principle, cited and copied from a public accounting reference. C. A sound industry practice, based on the internal auditor's knowledge and experience obtained during many engagement assignments within the organization. D. All of the answers represent an appropriate standard or criterion to support engagement observations, conclusions, and recommendations.

D. All of the answers represent an appropriate standard or criterion to support engagement observations, conclusions, and recommendations.

In confirming accounts receivable, an internal auditor decided to confirm customers' account balances rather than individual invoices. Which of the following most likely will be included with the organization confirmation letter? A. An auditor-prepared letter explaining that a nonresponse may cause an inference that the account balance is correct. B. An organization-prepared letter reminding the customer that a nonresponse will cause a second request to be sent. C. An auditor-prepared letter requesting the customer to supply missing and incorrect information directly to the client. D. An organization-prepared statement of account showing the details of the customer's account balance.

D. An organization-prepared statement of account showing the details of the customer's account balance.

Which of the following is true of benchmarking? A. Benchmarking is typically accomplished by comparing an organization's performance with the performance of its closest competitors. B. Benchmarking can be performed by using only qualitative comparisons. C. Benchmarking is normally limited to manufacturing operations and production processes. D. Benchmarking is accomplished by comparing an organization's performance to that of the best-performing organizations.

D. Benchmarking is accomplished by comparing an organization's performance to that of the best-performing organizations.

A manufacturer that wants to improve its staging process compares its procedures against the check-in process for a major airline. Which of the following tools is the manufacturer using? A. Total quality management. B. Statistical process control. C. Economic value added. D. Benchmarking.

D. Benchmarking.

The risk that an auditor's procedures will lead to the conclusion that a material misstatement does not exist in an account balance when, in fact, such misstatement does exist is A. Audit risk. B. Inherent risk. C. Control risk. D. Detection risk.

D. Detection risk.

As part of planning an engagement, the internal auditor in charge does all of the following except A. Determine the period covered. B. Conduct meetings with management responsible for the activity under review. C. Distribute reports from meetings with management. D. Determine to whom engagement results will be communicated.

D. Determine to whom engagement results will be communicated.

Fact pattern: A certified internal auditor is the chief audit executive for a large city and is planning the engagement work schedule for the next year. The city has a number of different funds, some that are restricted in use by government grants and some that require compliance reports to the government. One of the programs for which the city has received a grant is job retraining and placement. The grant specifies certain conditions a participant in the program must meet to be eligible for the funding. The internal auditors must determine the applicable laws and regulations. Which of the following procedures is the least effective in learning about the applicable laws and regulations? A. Make inquiries of the city's chief financial officer, legal counsel, or grant administrators. B. Review prior-year working papers and inquire of officials as to changes. C. Review applicable grant agreements. D. Discuss the matter with the board and make inquiries as to the nature of the requirements and the board's objectives for the engagement.

D. Discuss the matter with the board and make inquiries as to the nature of the requirements and the board's objectives for the engagement.

The Standards of The IIA apply to assurance services and consulting services performed by the internal audit activity (IAA). Which of the following is addressed only in a performance standard? A. Documentation. B. Planning considerations. C. Engagement objectives. D. Engagement supervision.

D. Engagement supervision.

In planning an assurance engagement, a survey could assist with all of the following except A. Obtaining engagement client comments and suggestions on control problems. B. Obtaining preliminary information on controls. C. Identifying areas for engagement emphasis. D. Evaluating the adequacy and effectiveness of controls.

D. Evaluating the adequacy and effectiveness of controls.

In violation of a company policy, Lowell Company erroneously capitalized the cost of painting its warehouse. The internal auditor examining Lowell's financial statements would most likely detect this error when A. Discussing capitalization policies with Lowell's controller. B. Examining maintenance expense accounts. C. Observing, during the physical inventory observation, that the warehouse had been painted. D. Examining the construction work orders supporting items capitalized during the year.

D. Examining the construction work orders supporting items capitalized during the year.

An internal auditor analyzes repairs and maintenance accounts primarily to obtain evidence in support of the classification assertion that all A. Noncapitalizable expenditures for repairs and maintenance have been recorded in the proper period. B. Expenditures for property and equipment have been recorded in the proper period. C. Noncapitalizable expenditures for repairs and maintenance have been properly charged to expense. D. Expenditures for property and equipment have not been charged to expense.

D. Expenditures for property and equipment have not been charged to expense.

In some countries, governmental units have established audit standards. For example, in the United States, the Government Accountability Office has developed standards for the conduct of governmental audits, particularly those that relate to compliance with government grants. In performing governmental grant compliance audits, the auditor should A. Be guided only by the governmental standards. B. Be guided only by The IIA Standards because they are more encompassing. C. Be guided by the more general standards that have been issued by the public accounting profession. D. Follow both The IIA Standards and any additional governmental standards.

D. Follow both The IIA Standards and any additional governmental standards.

During a preliminary survey of the accounts receivable function, an internal auditor discovered a potentially major control deficiency while preparing a flowchart. What immediate action should the internal auditor take regarding the weakness? A. Perform sufficient testing to determine its cause and effect. B. Report it to the level of management responsible for corrective action. C. Schedule a separate engagement to evaluate that segment of the accounts receivable function. D. Highlight the weakness to ensure that procedures to test it are included in the engagement work program.

D. Highlight the weakness to ensure that procedures to test it are included in the engagement work program.

Several members of an organization's senior management have questioned whether the internal audit activity should report to the newly established quality audit function as part of the total quality management process within the organization. The chief audit executive (CAE) has reviewed the quality audit standards and the programs that the quality audit manager has proposed. The CAE's response to senior management should include which of the following? A. Changing the applicable standards for internal auditing within the organization to provide compliance with quality audit standards. B. Changing the qualification requirements for new staff members to include quality audit experience. C. Estimating departmental cost savings that would result from the elimination of the internal audit activity. D. Identifying appropriate liaison activities with the quality audit function to ensure coordination of audit schedules and overall audit responsibilities.

D. Identifying appropriate liaison activities with the quality audit function to ensure coordination of audit schedules and overall audit responsibilities.

On the basis of audit evidence gathered and evaluated, an auditor decides to decrease the level of detection risk from that originally planned. Assuming the same planned audit risk level, the change in the planned detection risk most likely resulted from a(n) A. Decrease in the assessed control risk. B. Increase in materiality levels. C. Decrease in the assessed inherent risk. D. Increase in the assessed control risk.

D. Increase in the assessed control risk.

All of the following are acceptable criteria on which an internal audit may be based except A. Policies and procedures. B. Standards or guidelines. C. Control frameworks. D. Management cooperation with audit activities.

D. Management cooperation with audit activities.

Fact pattern: The chief audit executive (CAE) of a mid-sized internal audit activity was concerned that management might outsource the internal auditing function. Thus, the CAE adopted a very aggressive program to promote the internal audit activity within the organization. The CAE planned to present the results to senior management and the board and recommend modification of the internal audit activity's charter after using the new program. The following lists six actions the CAE took to promote a positive image within the organization: Engagement assignments concentrated on efficiency. The engagements focused solely on cost savings, and each engagement communication highlighted potential costs to be saved. Negative observations were omitted. The focus on efficiency was new, but the engagement clients seemed very happy. Drafts of all engagement communications were carefully reviewed with the engagement clients to get their input. Their comments were carefully considered when developing the final engagement communication. The information technology internal auditor participated as part of a development team to review the control procedures to be incorporated into a major computer application under development. Given limited resources, the engagement manager performed a risk assessment to establish engagement work schedule priorities. This was a marked departure from the previous approach of ensuring that all operations are evaluated on at least a 3-year interval. To save time, the CAE no longer required that a standard internal control questionnaire be completed for each engagement. When the internal auditors found that the engagement client had not developed specific criteria or data to evaluate operations, the internal auditors were instructed to perform research, develop specific criteria, review the criteria with the engagement client, and, if acceptable, use them to evaluate the engagement client's operations. If the engagement client disagreed with the criteria, a negotiation took place until acceptable criteria could be agreed upon. The engagement communication commented on the engagement client's operations in conjunction with the agreed-upon criteria. Is Action 5 inappropriate? A. Yes. Internal control should be evaluated on every engagement, but the internal control questionnaire is not the mandated approach to evaluate the controls. B. No. Internal auditors may omit necessary procedures if there is a time constraint. It is a matter of professional judgment. C. Yes. Internal control should be evaluated on every engagement, and the internal control questionnaire is the most efficient method to do so. D. No. Internal auditors are not required to fill out internal control questionnaires on every engagement.

D. No. Internal auditors are not required to fill out internal control questionnaires on every engagement.

An engagement to review payroll is least likely to include A. Tests of computations for gross and net wages. B. Comparison of payroll costs to budget. C. Tracing a sample of employee names to employment records in the personnel department. D. Observing the physical distribution of paychecks.

D. Observing the physical distribution of paychecks.

An audit committee should be designed to enhance the independence of both the internal and external auditing functions and to insulate these functions from undue management pressures. Using this criterion, audit committees should be composed of A. A rotating subcommittee of the board of directors or its equivalent. B. Only members from the relevant outside regulatory agencies. C. Members from all important constituencies, specifically including representatives from banking, labor, regulatory agencies, shareholders, and officers. D. Only external members of the board of directors or its equivalent.

D. Only external members of the board of directors or its equivalent.

A determination of cost savings is most likely to be an objective of a(n) A. Program-results engagement. B. Financial engagement. C. Compliance engagement. D. Operational engagement.

D. Operational engagement.

The internal auditor wishes to test the assertion that all claims paid by a medical insurer contain proper authorization and documentation, including but not limited to the validity of the claim from an approved physician and an indication that the claim complies with the claimant's policy. The most appropriate engagement procedure is to select a sample of A. All policyholders and examine all claims for the sampled items during the year to determine whether the claims were handled properly. B. Claims filed and trace to documentary information about authorization and other supporting documentation. C. Claims denied and determine that all claims denied were appropriate. The claims denied file is much smaller and the internal auditor can obtain greater coverage with the sample size. D. Paid claims from the claims (cash) disbursement file and vouch to documentary information about authorization and other supporting documentation.

D. Paid claims from the claims (cash) disbursement file and vouch to documentary information about authorization and other supporting documentation.

An approved audit plan for the internal audit activity is an essential part of A. Scheduling support for the external audit. B. Establishing standards for employee performance. C. Providing senior management with information about the quality of the internal audit activity's performance. D. Planning for the internal audit activity.

D. Planning for the internal audit activity.

Which of the following is most essential for guiding the internal audit staff? A. Quality program assessments. B. Position descriptions. C. Performance appraisals. D. Policies and procedures.

D. Policies and procedures.

A subsidiary president terminated a controller and hired a replacement without the required organizational approvals. Sales, cash flow, and profit statistics were then manipulated by the new controller and president via accelerated depreciation and sale of capital assets to obtain larger performance bonuses for the controller and the subsidiary president. An approach that might detect this fraudulent activity is A. Analysis of overall management control for segregation of duties. B. Required exit interviews for all terminated employees. C. Periodic changes of outside public accountants. D. Regular analytical review of operating divisions.

D. Regular analytical review of operating divisions.

Internal auditors are often called upon to either perform or assist the external auditor in performing a due diligence review. A due diligence review may be a(n) A. Review of interim financial statements as directed by an underwriting firm. B. Operational audit of a division of an organization to determine if divisional management is complying with laws and regulations. C. Review of operations as requested by the audit committee to determine whether the operations comply with audit committee and organizational policies. D. Review of financial statements and related disclosures in conjunction with a potential acquisition.

D. Review of financial statements and related disclosures in conjunction with a potential acquisition.

Which type of facilitated approach format begins by listing all possible barriers, obstacles, threats, and exposures that might prevent achieving an objective? A. Objective-based format. B. Control-based format. C. Process-based format. D. Risk-based format.

D. Risk-based format.

Organizations have multiple external (extended) business relationships (EBRs). They most likely involve A. Suppliers. B. Major customers. C. Regulators. D. Service providers.

D. Service providers.

All of the following are true regarding the process and methods of coordinating assurance activities except A. Assurance mapping connects significant risk categories and sources of assurance. B. In the combined assurance model, the internal audit activity coordinates with compliance activities. C. The formality of assurance activity coordination may vary with the size of the entity and any regulatory requirements. D. Sharing results with other providers violates the coordinating services agreement.

D. Sharing results with other providers violates the coordinating services agreement.

The primary difference between operational engagements and financial engagements is that, in the latter, the internal auditors A. Are not concerned with whether the client entity is generating information in compliance with financial accounting standards. B. Are seeking to help management use resources in the most effective manner possible. C. Can use analytical skills and tools that are not necessary in financial engagements. D. Start with the financial statements of the client entity and work backward to the basic processes involved in producing them.

D. Start with the financial statements of the client entity and work backward to the basic processes involved in producing them.

An internal auditor is more likely to detect duplicate vendor payments in a high exposure environment with certain internal control weaknesses by using which of the following procedures? A. Trend analysis. B. Monetary validity. C. Proportional analysis. D. Substantive testing.

D. Substantive testing.

Which of the following statements most accurately reflects the chief audit executive's responsibilities for internal audit resources? A. The CAE is responsible for ensuring that audit coverage is based on the periodic skills assessment. B. The CAE is responsible for evaluating the detailed summary of audit resources presented by management to the board. C. The CAE is not responsible for such human resource functions as evaluation and development. D. The CAE is responsible for communicating resource needs to the board but has no explicit responsibility for administering the organization's compensation program.

D. The CAE is responsible for communicating resource needs to the board but has no explicit responsibility for administering the organization's compensation program.

Which of the following is a characteristic of business process reengineering? A. Gradual, incremental streamlining of existing procedures. B. The movement of manual processes to computers. C. A change in the nature of the business itself. D. The bottom-up revision of the way the organization carries out a particular business process.

D. The bottom-up revision of the way the organization carries out a particular business process.

Which of the following is an effective tool for uncovering unethical or illegal activity in an organization? A. The screening of applicants. B. The ethics interview. C. The background check. D. The ethics questionnaire.

D. The ethics questionnaire.

Which of the following is not an appropriate member of an audit committee? A. The vice president of the local bank used by the organization. B. An academic specializing in business administration. C. A retired executive of a firm that had been associated with the organization. D. The organization's vice president of operations.

D. The organization's vice president of operations.

Numerous environmental laws and regulations have recently changed. Senior management has asked the chief audit executive to perform an environmental audit to be completed as soon as possible. The internal audit activity currently is performing an operational audit. As a result, the chief audit executive must make difficult decisions about resource allocation. Which of the following is the least significant issue in determining whether to reallocate audit resources? A. The potential fraud discovered during the operational audit. B. Potential cost to the organization for noncompliance with the new environmental laws and regulations. C. The knowledge, skills, and competencies of the internal audit staff. D. The results from the prior financial audits.

D. The results from the prior financial audits.

In a financial statement audit, inherent risk is evaluated to help an auditor assess which of the following? A. The internal audit department's objectivity in reporting to the audit committee a material misstatement of a financial statement assertion it detects. B. The risk that the internal control system will not detect a material misstatement of a financial statement assertion. C. The risk that the audit procedures implemented will not detect a material misstatement of a financial statement assertion. D. The susceptibility of a financial statement assertion to a material misstatement before consideration of related controls.

D. The susceptibility of a financial statement assertion to a material misstatement before consideration of related controls.

According to the International Professional Practices Framework, internal audit resources are effectively deployed when A. The internal audit staff has the necessary attributes for the planned activities. B. The resources needed to accomplish the plan are adequate. C. There are more opportunities to achieve operating benefits for the engagement client. D. They are used in a way that optimizes the achievement of the approved plan.

D. They are used in a way that optimizes the achievement of the approved plan.

Which of the following statements is true with respect to a time budget for an internal audit engagement? A. Requests for time budget adjustments should be approved by the audit committee. B. Time budgets should be strictly adhered to, regardless of circumstances. C. Time budgets should be used for financial audits, but not for operational audits. D. Time budgets should normally be prepared in terms of hours or days.

D. Time budgets should normally be prepared in terms of hours or days.

In most organizations, the rapidly expanding scope of internal auditing responsibilities requires continual training. What is the main purpose of such a training program? A. To comply with continuing education requirements of professional organizations. B. To use slack periods in engagement scheduling. C. To help individuals to achieve personal career goals. D. To achieve both individual and organizational goals.

D. To achieve both individual and organizational goals.

The internal auditor for ABC Corporation has received a special request from management. The internal auditor believes that the objectives that should be pursued go beyond those requested by management. What should the internal auditor do? A. Refuse to accept the engagement unless management can be persuaded to include the additional objectives in the consulting engagement. B. Include the objectives that are necessary in the current consulting engagement and inform management in the final communication of the engagement results. C. Document the fact that the objectives were not pursued and disclose that observation to the audit committee in a formal report. D. Try to persuade management to include the additional objectives in the consulting engagement.

D. Try to persuade management to include the additional objectives in the consulting engagement.

An internal auditor has been asked to review the treasury department's compliance with corporate policy related to the use of forward trading to manage currency valuation risk. The auditor finds no related policies in the corporate policy manual but does discover that the department is following a policy developed by the company's bank. Which of the following would be the most appropriate response from the auditor? A. Withdraw from the audit engagement, because there is nothing to audit due to the lack of a corporate policy. B. Perform no further audit work and report the lack of a corporate policy as an audit observation. C. Postpone the audit engagement until a corporate policy can be established. D. Use the bank's policy as the audit criteria and determine whether formal adoption should be recommended in the engagement final communication.

D. Use the bank's policy as the audit criteria and determine whether formal adoption should be recommended in the engagement final communication.

The chief audit executive set up a computerized spreadsheet to facilitate the risk assessment process involving a number of different divisions in the organization. The spreadsheet included the following factors: Pressure on divisional management to meet profit goals Complexity of operations Competence of divisional personnel The monetary amount of subjectively influenced accounts in the division, such as accounts in which management's judgment can affect the expense, e.g., postretirement benefits The CAE used a group meeting of internal audit managers to reach a consensus on the competence of divisional personnel. Other factors were assessed as high, medium, or low by either the CAE or an internal audit manager who had performed an engagement at the division. The CAE assigned a weight ranging from 0.5 to 1.0 to each factor and then computed a composite risk score. Which statement is true? A. The risk analysis is not appropriate because it mixes both quantitative and qualitative factors, thereby making expected value calculations impossible. B. Assessing factors at discrete levels such as high, medium, and low is inappropriate for the risk assessment process because the ratings are not quantifiable. C. The weighting is subjective and should have been determined through a process such as multiple-regression analysis. D. Using a subjective group consensus to assess personnel competence is appropriate.

D. Using a subjective group consensus to assess personnel competence is appropriate.

During the process of confirming receivables as of December 31, Year 1, a positive confirmation was returned indicating the "balance owed as of December 31 was paid on January 9, Year 2." The internal auditor would most likely A. Determine whether any changes in the account occurred between January 1 and January 9, Year 2. B. Determine whether a customary trade discount was taken by the customer. C. Reconfirm the zero balance as of January 10, Year 2. D. Verify that the amount was received.

D. Verify that the amount was received.

Which of the following privacy terms is matched with an accurate example of the term? Term Example A. Term Privacy of space Example Freedom from surveillance B. Term Privacy of information Example Freedom from monitoring C. Term Personal privacy Example Freedom from monitoring D. Term Privacy of communication Example Freedom from surveillance

A. Term Privacy of space Example Freedom from surveillance

A manufacturer uses a materials requirements planning (MRP) system to track inventory, orders, and raw material requirements. A preliminary audit assessment indicates that the organization's inventory is understated. Using audit software, what conditions should the auditor search for in the MRP database to support this hypothesis? 1. Item cost set at zero 2. Negative quantities on hand 3. Order quantity exceeding requirements 4. Inventory lead times exceeding delivery schedule A. 1 and 2 only. B. 1 and 4 only. C. 2 and 4 only. D. 3 and 4 only.

A. 1 and 2 only.

A United States organization plans to expand operations to Gambia. Which of the following are plausible assurance engagement objectives for an internal audit of the human resources department? 1. Evaluate the clarity and transparency of the design of global compensation and benefit systems. 2. Identify methods of improving communications with assignees, line management, and leadership. 3. Assess compliance with applicable requirements for visas and work permits. 4. Provide consultation to potential assignees and line management on terms and conditions of the internal assignment. A. 1 and 3 only. B. 2 and 4 only. C. 3 and 4 only. D. 1, 2, and 4 only.

A. 1 and 3 only.

The element(s) of a control self-assessment (CSA) performed using one of the facilitated team workshop approaches include(s) 1. Treating participating employees as process owners. 2. Taking a simple yes/no survey of employees regarding risks and controls. 3. Interviewing employees separately in the field. A. 1 only. B. 2 only. C. 2 and 3. D. 1, 2, and 3.

A. 1 only.

An employee of an insurer processed a fraudulent policy loan application for an amount less than the established level requiring supervisory review. The employee then obtained the check and cashed it by forging the endorsement. To prevent the loan's appearance on a subsequent policyholder statement, the loan amount was transferred to a "suspense" account. Which of the following should expose this situation at the earliest date? A. A computer report identifying unusual entries to the suspense account. B. The use of prenumbered checks that are periodically accounted for. C. An annual engagement performed by the internal audit activity. D. Regular reconciliation of the "suspense" account performed by an independent employee.

A. A computer report identifying unusual entries to the suspense account.

During a preliminary survey, an auditor found that several accounts payable vouchers for major suppliers required adjustments for duplicate payment of prior invoices. This would indicate A. A need for additional testing to determine related controls and the current exposure to duplicate payments made to suppliers. B. The possibility of unrecorded liabilities for the amount of the overpayments. C. Insufficient controls in the receiving area to ensure timely notice to the accounts payable area that goods have been received and inspected. D. The existence of a sophisticated accounts payable system that correlates overpayments to open invoices and therefore requires no further audit concern.

A. A need for additional testing to determine related controls and the current exposure to duplicate payments made to suppliers.

A chief audit executive most likely uses risk assessment for audit planning because it provides A. A systematic process for assessing and integrating professional judgment about probable adverse conditions. B. A listing of potentially adverse effects on the organization. C. A list of auditable activities in the organization. D. The probability that an event or action may adversely affect the organization.

A. A systematic process for assessing and integrating professional judgment about probable adverse conditions.

For an upcoming engagement, an internal auditor's objective is to determine whether costs are both documented and reasonable. This is most likely an engagement involving A. Advertising agency billings. B. Allowance for doubtful accounts. C. Asset disposals. D. Accounts payable.

A. Advertising agency billings.

A program-results engagement is most likely to be performed on A. An activity not part of normal operations. B. The purchasing and receiving departments. C. Safety practices and scrap handling. D. Distribution of services and materials.

A. An activity not part of normal operations.

An assurance engagement in the quality control department is being planned. Which of the following is least likely to be used in the preparation of a preliminary survey questionnaire? A. An analysis of quality control documents. B. The permanent engagement file. C. The prior engagement communications. D. Management's charter for the quality control department.

A. An analysis of quality control documents.

An organization is considering purchasing a small toxic waste disposal business. The internal auditors are part of the team doing a due diligence review for the acquisition. The scope of the internal auditors' work will most likely not include A. An evaluation of the merit of lawsuits currently filed against the acquiree. B. A review of the acquiree's procedures for acceptance of waste material and comparison with legal requirements. C. Analysis of the acquiree's compliance with, and disclosure of, loan covenants. D. Assessment of the efficiency of the operations of the acquiree.

A. An evaluation of the merit of lawsuits currently filed against the acquiree.

In managing internal audit resources, the CAE considers all of the following except A. Benchmarking. B. Succession planning. C. Staff evaluation and development. D. Resourcing needs.

A. Benchmarking.

According to the Standards, internal auditors determine appropriate and sufficient resources to achieve engagement objectives based on an evaluation of all of the following except A. Budgetary control of the engagement. B. Complexity of each engagement. C. Time constraints for each engagement. D. Available resources for each engagement.

A. Budgetary control of the engagement.

Which of the following statements about control self-assessment (CSA) is false? A. CSA is usually an informal and undocumented process. B. In its purest form, CSA integrates business objectives and risks with control processes. C. CSA is also known as control/risk self-assessment. D. Most implemented CSA programs share some key features and goals.

A. CSA is usually an informal and undocumented process.

Audit risk at the assertion level consists of inherent risk, control risk, and detection risk. Which of the following statements is true? A. Cash has a greater inherent risk than an inventory of coal because it is more susceptible to theft. B. The risk that material misstatement will not be timely prevented or detected by internal control can be reduced to zero by effective controls. C. Detection risk is a function of the efficiency of an auditing procedure. D. The existing levels of inherent risk, control risk, and detection risk can be changed at the discretion of the auditor.

A. Cash has a greater inherent risk than an inventory of coal because it is more susceptible to theft.

The chief audit executive's (CAE) responsibility for assessing and reporting on control processes includes A. Communicating to senior management and the board an annual judgment about internal control. B. Overseeing the establishment of internal control processes. C. Maintaining the organization's governance processes. D. Arriving at a single assessment based solely on the work of the internal audit activity.

A. Communicating to senior management and the board an annual judgment about internal control.

In an engagement to review a not-for-profit organization's special revenue fund, the primary engagement objective is to determine whether the organization A. Complied with existing fund requirements and performed specified activities. B. Managed its resources economically and efficiently. C. Prepared its financial statements in accordance with accounting principles generally accepted in its country. D. Applies the funds in a way that would benefit the greatest number of people.

A. Complied with existing fund requirements and performed specified activities.

Management is evaluating the need for an environmental audit program. Which one of the following should not be included as an overall program objective? A. Conduct site assessments at all waste-producing facilities. B. Verify organizational compliance with all environmental laws. C. Evaluate waste minimization opportunities. D. Ensure management systems are adequate to minimize future environmental risks.

A. Conduct site assessments at all waste-producing facilities.

To identify the amount of obsolete inventory that may exist in an organization, an internal auditor probably should collect information using all of the following procedures except A. Confirmation. B. Scanning. C. Recomputation. D. Analytical review.

A. Confirmation.

If a department outside of the internal audit activity is responsible for reviewing a function or process, the internal auditors should A. Consider the work of the other department when assessing the function or process. B. Ignore the work of the other department and proceed with an independent audit. C. Reduce the scope of the audit since the work has already been performed by the other department. D. Yield the responsibility for assessing the function or process to the other department.

A. Consider the work of the other department when assessing the function or process.

With regard to providing an assurance service for the organization's privacy framework, the internal audit activity assesses the adequacy of risk identification and controls. The internal audit activity also A. Considers practices in relevant jurisdictions. B. Confirms to the board that information security is the IAA's responsibility. C. Performs a consulting engagement to provide advice on information security protocols. D. Devises and implements controls.

A. Considers practices in relevant jurisdictions.

Updating the audit universe is useful in developing the internal audit plan. The audit universe A. Consists of all possible audits. B. Reflects only past organizational strategies. C. May not overlap with the organization's strategic plan. D. Is typically updated every 5 years.

A. Consists of all possible audits.

A company has computerized sales and cash receipts journals. The computer programs for these journals have been properly debugged. The auditor discovered that the total of the accounts receivable subsidiary accounts differs materially from the accounts receivable control account. This discrepancy could indicate A. Credit memoranda being improperly recorded. B. Lapping of receivables. C. Receivables not being properly aged. D. Statements being intercepted prior to mailing.

A. Credit memoranda being improperly recorded.

Which of the following is not likely to be an assurance engagement objective related to auditing governance activities? A. Determine customer satisfaction with shareholder communications. B. Determine the operating effectiveness of the whistleblower process. C. Evaluate the design adequacy of board education and training. D. Assess compliance with ethics policies.

A. Determine customer satisfaction with shareholder communications.

In evaluating the effectiveness and efficiency with which resources are employed, an internal auditor is responsible for A. Determining the extent to which adequate operating criteria have been established. B. Verifying the existence of assets. C. Reviewing the reliability of operating information. D. Verifying the accuracy of asset valuation.

A. Determining the extent to which adequate operating criteria have been established.

The preliminary survey indicates that severe staff reductions at the engagement location have resulted in extensive amounts of overtime among accounting staff. Department members are visibly stressed and very vocal about the effects of the cutbacks. Accounting payrolls are nearly equal to prior years, and many key controls, such as segregation of duties, are no longer in place. The accounting supervisor now performs all operations within the cash receipts and posting process and has no time to review and approve transactions generated by the remaining members of the department. Journal entries for the last 6 months since the staff reductions show increasing numbers of prior-month adjustments and corrections, including revenues, cost of sales, and accruals that had been misstated or forgotten during month-end closing activity. The internal auditor should A. Discuss these observations with management of the internal audit activity to determine whether further work would be an efficient use of internal auditing resources at this time. B. Proceed with the scheduled engagement but add personnel based on the expected number of observations and anticipated lack of assistance from local accounting management. C. Research temporary help agencies and evaluate the cost and benefit of outsourcing needed services. D. Suspend further engagement work and issue the final communication of results because the conclusions are obvious.

A. Discuss these observations with management of the internal audit activity to determine whether further work would be an efficient use of internal auditing resources at this time.

In the planning phase, the scope of an internal audit engagement is defined by the A. Engagement objectives. B. Scheduling and time estimates. C. Preliminary survey. D. Engagement work program.

A. Engagement objectives.

Which of the following is part of the board's role in protecting against privacy threats? A. Establishing a privacy framework. B. Identifying the information gathered by the organization that is deemed personal or private. C. Identifying the methods used to collect information. D. Determining whether the use of the information collected is in accordance with its intended use and the laws.

A. Establishing a privacy framework.

An investment portfolio manager has the authority to use financial derivatives to hedge transactions but is not supposed to take speculative positions. However, the manager launches a scheme that includes (1) taking a position larger than required by the hedge, (2) putting the speculative gains in a suspense account, and (3) transferring the funds to a nonexistent broker and from there to a personal account. Which of the following engagement procedures is least effective in detecting this fraud? A. Examine individual trades to determine whether the trades violate the authorization limit for the manager. B. Sample individual trades and determine the exact matching of a hedge. Schedule and investigate all differences. C. Sample all debits to the suspense account and examine their disposition. D. Sample fund transfers to brokers and determine if the brokers are on the organization's authorized list for transactions.

A. Examine individual trades to determine whether the trades violate the authorization limit for the manager.

If an auditor's preliminary evaluation of internal controls results in an observation that controls may be inadequate, the next step is to A. Expand audit work prior to the preparation of an engagement final communication. B. Prepare a flowchart depicting the internal control system. C. Note an exception in the engagement final communication if losses have occurred. D. Implement the desired controls.

A. Expand audit work prior to the preparation of an engagement final communication.

The acceptable level of detection risk is inversely related to the A. Extent of engagement procedures performed. B. Risk of misapplying auditing procedures. C. Preliminary judgment about materiality levels. D. Risk of failing to discover material misstatements.

A. Extent of engagement procedures performed.

After using the same public accounting firm for several years, the board of directors retained another public accounting firm to perform the annual financial audit in order to reduce the annual audit fee. The new firm has now proposed a one-time engagement relating to the cost-effectiveness of the various operations of the business. The chief audit executive has been asked to advise management in making a decision on the proposal. An argument can be made that the internal audit activity is better able to perform such an engagement because A. External auditors may not possess the same depth of understanding of the organization as the internal auditors. B. Internal auditors are required to be objective in performing engagements. C. Engagement procedures used by internal auditors are different from those used by external auditors. D. Internal auditors will not be vitally concerned with fraud and waste.

A. External auditors may not possess the same depth of understanding of the organization as the internal auditors.

Johnny Hagerts, the chief audit executive of Booster, Inc., is having a meeting with senior management about the status of the internal audit. In this meeting, Mr. Hagerts should provide assurance to management about which of the following? A. Governance, risk management, and control. B. Sufficiency of internal audit staff. C. The time schedule of the engagement. D. The frequency and nature of reports.

A. Governance, risk management, and control.

What is the most accurate term for the procedures used by the board to oversee activities performed to achieve organizational objectives? A. Governance. B. Control. C. Risk management. D. Monitoring.

A. Governance.

An advantage of conducting environmental audits under the direction of the internal audit activity is that A. Independence and authority are already in place. B. Technical expertise is more readily available. C. The financial aspects are de-emphasized. D. Internal auditing work products are confidential.

A. Independence and authority are already in place.

An internal auditor has set an engagement objective of ascertaining compliance with a city ordinance forbidding city purchasing from vendors affiliated with elected city officials. Which of the following engagement techniques will best meet this objective? A. Inspection of documents. B. Observation. C. Inquiry. D. Analytical review.

A. Inspection of documents.

Which of the following statements is false with respect to information security? A. Internal auditors should determine that senior management and the board, audit committee, or other governing body have a clear understanding that information reliability and integrity is the responsibility of the internal audit activity. B. The chief audit executive should determine that the internal audit activity possesses, or has access to, competent auditing resources to evaluate information security and associated risk exposures. C. Internal auditors should periodically assess the organization's information security practices and recommend, as appropriate, enhancements to, or implementation of, new controls and safeguards. D. Internal auditors should assess the effectiveness of preventive, detective, and mitigative measures against past attacks, as deemed appropriate, and future attempts or incidents deemed likely to occur.

A. Internal auditors should determine that senior management and the board, audit committee, or other governing body have a clear understanding that information reliability and integrity is the responsibility of the internal audit activity.

Although all the current members of an internal audit activity have good records of performance, the manager is not sure if any of the members are ready to assume a management role. Which of the following is an advantage of bringing in an outsider rather than promoting from within? A. Management training costs are reduced when a qualified outsider is hired. B. The manager can be sure that the new position will be filled by a competent employee. C. Bringing in an outsider is a less expensive alternative than promoting from within. D. The "modeling" effect is strengthened by bringing in a new role model.

A. Management training costs are reduced when a qualified outsider is hired.

What is the role of a chief audit executive (CAE) with regard to an inspection by a regulator? A. Meet with the regulator before and after the inspection to provide relevant information or receive advice on necessary compliance. B. Meet with the regulator after the inspection to dispute any negative findings about compliance. C. Tour the facility with the regulator to ensure that no problems are uncovered. D. Meet with specific managers to protect proprietary information.

A. Meet with the regulator before and after the inspection to provide relevant information or receive advice on necessary compliance.

An internal auditor discovers during an engagement involving the entity's environmental, health, and safety (EHS) department that department personnel are poorly informed about legal issues resulting from discharging waste into municipal water sources. The EHS function is small. Which of the following is the best course of action for the auditor to take? A. Note the control weakness and perform additional procedures to help determine its potential effects. B. Arrange for a training session for the EHS staff with experts in the field of wastewater legal issues. C. Immediately narrow the scope of the engagement to examine wastewater discharge. D. Report possible violations to the relevant regulatory authority.

A. Note the control weakness and perform additional procedures to help determine its potential effects.

Controls should be designed to ensure that A. Operations are performed efficiently. B. Management's plans have not been circumvented by worker collusion. C. The internal audit activity's guidance and oversight of management's performance is accomplished economically and efficiently. D. Management's planning, organizing, and directing processes are properly evaluated.

A. Operations are performed efficiently.

Controls should be designed to provide reasonable assurance that A. Organizational objectives will be achieved economically and efficiently. B. Management's plans have not been circumvented by worker collusion. C. The internal audit activity's guidance and oversight of management's performance is accomplished economically and efficiently. D. Management's planning, organizing, and directing processes are properly evaluated.

A. Organizational objectives will be achieved economically and efficiently.

In selecting an instructional strategy for developing internal audit staff, a chief audit executive begins by reviewing A. Organizational objectives. B. Learning content. C. Learners' readiness. D. Budget constraints.

A. Organizational objectives.

Which of the following is a false statement about the relationship between internal auditors and external auditors? A. Oversight of the work of external auditors is the responsibility of the chief audit executive. B. Sufficient meetings are scheduled between internal and external auditors to ensure timely and efficient completion of the work. C. Internal and external auditors may exchange engagement communications and management letters. D. Internal auditors may provide engagement work programs and working papers to external auditors.

A. Oversight of the work of external auditors is the responsibility of the chief audit executive.

Which of the following is not an objective of an environmental audit program? A. Perform walkthroughs of all processes that contain identified environmental risks. B. Verify organizational compliance with all environmental laws. C. Review the reasonableness and likelihood of contingent liabilities accrued for environmental remediation. D. Ensure management systems are adequate to minimize future environmental risks.

A. Perform walkthroughs of all processes that contain identified environmental risks.

A page from an internal auditor's workpapers contains notes made in the prior period. They specify which controls are relevant in the current period and which controls will soon be obsolete. The notes relate to A. Preliminary survey. B. Review for adequacy of control processes. C. Review for effectiveness of control processes. D. Review of results.

A. Preliminary survey.

During which phase of the engagement does the internal auditor identify the objectives and related controls of the activity being examined? A. Preliminary survey. B. Staff selection. C. Work program preparation. D. Final communication of results.

A. Preliminary survey.

Fact pattern: You are an internal auditing supervisor who is reviewing the working papers of a staff internal auditor's overall examination of the firm's sales function. The pages are not numbered or cross-referenced. Furthermore, the working papers were dropped and reassembled at random before they were brought to you. You decide to put the working papers in the proper order according to the Standards. The first stage of this activity is to identify each page as a part of (1) the preliminary survey, (2) the review of the adequacy of control processes, (3) the review for effectiveness of control processes, or (4) the review of results. The second page the supervisor selects documents an interview with a salesperson discussing the overall sales cycle. This page belongs with which activity? A. Preliminary survey. B. Review for adequacy of control processes. C. Review for effectiveness of control processes. D. Review of results.

A. Preliminary survey.

The established scope of the engagement must be sufficient to satisfy the objectives of the engagement. When developing the objectives of the engagement, the internal auditor considers the A. Probability of significant noncompliance. B. Information included in the engagement work program. C. Results of engagement procedures. D. Resources required.

A. Probability of significant noncompliance.

Fact pattern: You are the chief audit executive of a parent organization that has foreign subsidiaries. Independent external audits performed for the parent are not conducted by the same firm that conducts the foreign subsidiary audits. Because the internal audit activity occasionally provides direct assistance to both external firms, you have copies of audit programs and selected working papers produced by each firm. The foreign subsidiary's external audit firm wants to rely on an audit of a function at the parent organization. The audit was conducted by the internal audit activity. To place reliance on the work performed, the foreign subsidiary's auditors have requested copies of the working papers. What is the most appropriate response to the foreign subsidiary's auditors? A. Provide copies of the working papers. B. Ask the parent's audit firm if it is appropriate to release the working papers. C. Ask the board for permission to release the working papers. D. Refuse to provide the working papers under any circumstances.

A. Provide copies of the working papers.

The chief audit executive routinely reports to the board as part of the board meeting agenda each quarter. Senior management has asked to review this presentation before each board meeting so that any issues or questions can be discussed beforehand. The CAE needs to A. Provide the report to senior management as requested and discuss any issues that may require action to be taken. B. Withhold disclosure of the report to senior management because such matters are the sole province of the board. C. Disclose to the board only those matters in the report that pertain to expenditures and financial budgets of the internal audit activity. D. Provide information to senior management that pertains only to completed engagements and observations available in published engagement communications.

A. Provide the report to senior management as requested and discuss any issues that may require action to be taken.

Documentation required to plan an internal audit engagement includes information that A. Resources needed to complete the engagement were considered. B. Planned engagement work will be completed on a timely basis. C. Intended engagement observations have been clearly identified. D. Internal audit activity resources are efficiently and effectively employed.

A. Resources needed to complete the engagement were considered.

If the objective of an auditor's test of details is to detect the overstatement of sales, the auditor should trace transactions from the A. Sales journal to the shipping documents. B. Shipping documents to the cash receipts journal. C. Cash receipts journal to the customer's purchase orders. D. Customer's purchase orders to the sales journal.

A. Sales journal to the shipping documents.

Fact pattern: The chief audit executive (CAE) of a mid-sized internal audit activity was concerned that management might outsource the internal auditing function. Thus, the CAE adopted a very aggressive program to promote the internal audit activity within the organization. The CAE planned to present the results to senior management and the board and recommend modification of the internal audit activity's charter after using the new program. The following lists six actions the CAE took to promote a positive image within the organization: Engagement assignments concentrated on efficiency. The engagements focused solely on cost savings, and each engagement communication highlighted potential costs to be saved. Negative observations were omitted. The focus on efficiency was new, but the engagement clients seemed very happy. Drafts of all engagement communications were carefully reviewed with the engagement clients to get their input. Their comments were carefully considered when developing the final engagement communication. The information technology internal auditor participated as part of a development team to review the control procedures to be incorporated into a major computer application under development. Given limited resources, the engagement manager performed a risk assessment to establish engagement work schedule priorities. This was a marked departure from the previous approach of ensuring that all operations are evaluated on at least a 3-year interval. To save time, the CAE no longer required that a standard internal control questionnaire be completed for each engagement. When the internal auditors found that the engagement client had not developed specific criteria or data to evaluate operations, the internal auditors were instructed to perform research, develop specific criteria, review the criteria with the engagement client, and, if acceptable, use them to evaluate the engagement client's operations. If the engagement client disagreed with the criteria, a negotiation took place until acceptable criteria could be agreed upon. The engagement communication commented on the engagement client's operations in conjunction with the agreed-upon criteria. Regarding Action 6, which of the following elements of the action most likely would have rendered it inappropriate if omitted? A. Seek agreement with the client about the criteria. B. Developing a set of criteria to present to the engagement client as a basis for evaluating the engagement client's operations. C. Commenting on the agreed-upon criteria. D. All of the answers are correct.

A. Seek agreement with the client about the criteria.

The chief audit executive for a large decentralized organization has developed a manual containing comprehensive detailed written procedures as a guide for the decentralized engagement work groups, each of which has 20 to 30 internal auditors. The organization recently acquired a small organization that has an internal audit activity consisting of a supervisor and two staff personnel. Which of the following actions is the most practical in providing administrative guidance for this new internal audit activity? A. Select key procedures from the manual and use informal supervisory direction for other engagement management issues. B. Use informal supervisory direction for engagement management issues. C. Use the already developed manual. D. Adopt the administrative procedures being followed by the internal auditors of the acquired organization.

A. Select key procedures from the manual and use informal supervisory direction for other engagement management issues.

Who reviews and approves a summary of the internal audit plan? A. Senior management and the board. B. The audit committee and the board. C. Senior management only. D. The chief audit executive (CAE) only.

A. Senior management and the board.

Risk assessment is a systematic process for assessing and integrating professional judgments about probable adverse conditions or events. Which of the following statements reflects the appropriate action for the chief audit executive to take? A. The CAE should generally assign engagement priorities to activities with higher risks. B. The CAE should restrict the number of sources of information used in the risk assessment process. C. Work schedule priorities should be established to lead the CAE in the risk assessment process. D. The risk assessment process should be conducted at least every 3 to 5 years.

A. The CAE should generally assign engagement priorities to activities with higher risks.

Which of the following is not a requirement of risk-based audit planning? A. The chief audit executive consults with external auditors. B. The risk-based plan considers the organization's strategies and objectives. C. The risk-based plan is adjusted for changes in the organization's business. D. To determine the priorities of the internal audit activity, a risk-based plan must be established.

A. The chief audit executive consults with external auditors.

One of the main reasons total quality management (TQM) can be used as a strategic weapon is that A. The cumulative improvement from a company's TQM efforts cannot readily be copied by competitors. B. Introducing new products can lure customers away from competitors. C. Reduced costs associated with better quality can support higher shareholder dividends. D. TQM provides a comprehensive planning process for a business.

A. The cumulative improvement from a company's TQM efforts cannot readily be copied by competitors.

The internal auditor is considering making a risk analysis as a basis for determining the areas of the organization where engagements should be performed. Which one of the following statements is true regarding risk analysis? A. The extent to which management judgments are required in an area could serve as a risk factor in assisting the internal auditor in making a comparative risk analysis. B. The highest risk assessment should always be assigned to the area with the largest potential loss. C. The highest risk assessment should always be assigned to the area with highest probability of occurrence. D. Risk analysis must be reduced to quantitative terms in order to provide meaningful comparisons across an organization.

A. The extent to which management judgments are required in an area could serve as a risk factor in assisting the internal auditor in making a comparative risk analysis.

Internal auditors must make a preliminary assessment of risks when conducting an assurance engagement. This assessment may involve quantitative (objective) and subjective factors. The least subjective factor is A. The organization's recognized losses on derivatives. B. The auditor's assessment of management responses. C. Changes in the auditee's business forecast. D. The evaluation of internal control.

A. The organization's recognized losses on derivatives.

An organization is considering purchasing a commercial property. Because of the location of the property and the known recent history of activities on the property, management has asked the internal audit activity, in cooperation with legal counsel, to provide a preliminary identification of any environmental liability. The strongest reason supporting management's decision to request such an investigation is A. The potential for future liability may outweigh any advantages achieved by obtaining the property. B. Management will be able to pay a lower price for the property if environmental contamination can be identified. C. The current owner would be required by law to clean up all identified contamination before the sale is closed. D. Regulatory agencies require a purchaser to identify and disclose all actual and potential instances of contamination.

A. The potential for future liability may outweigh any advantages achieved by obtaining the property.

Which of the following is a definition of control risk? A. The risk that a material misstatement will not be prevented or detected on a timely basis by the client's internal controls. B. The risk that the auditor will not detect a material misstatement. C. The risk that the auditor's assessment of internal controls will be at less than the maximum level. D. The susceptibility of material misstatement assuming there are no related internal control policies or procedures.

A. The risk that a material misstatement will not be prevented or detected on a timely basis by the client's internal controls.

Staff members of the internal audit activity should be assigned to engagements and training projects that will enable them to develop their potential. Which of the following should be the most important consideration in making assignments that will allow staff members to develop properly? A. The skills and experience levels of individual auditors. B. Specific training requirements imposed by the Standards. C. The importance of giving all staff members extensive supervisory experience. D. Special interests of individual staff members.

A. The skills and experience levels of individual auditors.

When assigning individual staff members to actual engagements, internal auditing managers are faced with a number of important considerations related to needs, abilities, and skills. Which of the following is the least appropriate criterion for assigning a staff internal auditor to a specific engagement? A. The staff internal auditor's desire for training in the area. B. The complexity of the engagement. C. The experience level of the internal auditor. D. Special skills possessed by the staff internal auditor.

A. The staff internal auditor's desire for training in the area.

Which of the following is the best reason for the chief audit executive to consider the strategic plan in developing the annual audit plan? A. To ensure that the internal audit plan supports the overall business objectives. B. To ensure that the internal audit plan will be approved by senior management. C. To make recommendations to improve the strategic plan. D. To emphasize the importance of the internal audit function.

A. To ensure that the internal audit plan supports the overall business objectives.

When are governance, risk management, and control processes considered adequate? A. When management has planned and designed them to provide reasonable assurance of achieving the organization's objectives efficiently and economically. B. When management has planned and designed them to provide absolute assurance of achieving the organization's objectives efficiently and economically. C. When the internal audit activity has planned and designed them to provide reasonable assurance of achieving the organization's objectives efficiently and economically. D. When the company is profitable.

A. When management has planned and designed them to provide reasonable assurance of achieving the organization's objectives efficiently and economically.

At a meeting with engagement managers, the chief audit executive is allocating the engagement work schedule for next year's plan. Which of the following methods will ensure that each manager receives an appropriate share of both the work schedule and internal audit activity resources? A. Work is assigned to each manager based on risk and skill analysis. B. Each of the managers selects the individual assignments desired, based on preferences for the area and the management personnel involved. C. Each manager chooses assignment preferences based on the total staff hours that are currently available to each manager. D. The full list of scheduled engagements is published for the staff, and work assignments are made based on career interests and travel requirements.

A. Work is assigned to each manager based on risk and skill analysis.


Kaugnay na mga set ng pag-aaral

Chapter 3: Understanding Diverse Literacy Needs

View Set

Quiz: Business Uses of Life Insurance

View Set